Jump to content

Ohm's Law and Watt's Law questions.


ThatOtherChigga #Mack

Recommended Posts

Question #1: A 1/4 watt resistor is connected to a six volt source. How much current is required to reach the maximum 1/4 rating of the resistor.

 

So is it 1/4*6 = C or is it a different equation? (Current is measured in Ohms right?)

 

Question #2: A 1/2 watt 1200 ohm resistor is connected to a power source. What is the maximum voltage that can be applied to math its power rating?

 

So what's the equation, and why?

 

Question #3: A circuit design specification states that all carbon composite resistors used shall not exceed 50% of their rated wattage. What is the maximum current a 1.2 kiloOhm, 1/4 watt carbon composite resistor can safely carry and meet the specifications?

Equation and why?

 

Ohm's Law: E=I*R

Watt's law: P=I*E

 

Don't forget stuff like E=P/I, I=E/R, etc.

Link to comment
Share on other sites

#1- Current is measured in Amps. Resistance is measured in Ohms.

#2 and #3- You'll have to use both equations together. Power, voltage, current, and resistance are all connected to each other by the two laws.

Link to comment
Share on other sites

Here's what I got:

#1 I= P/V
      = 0.25W/6V
      = 0.0417A
   
#2 V^2= PR 
          = 0.25W*1200ohm   =0.5W*1200ohm
          = 300V^2                  =600V^2
     V= 17.3V                  V =24.5V
 
#3 I^2= P/R 
         = 0.125W / 1200ohm
         = 0.000104 A^2
     I= 0.01A
Link to comment
Share on other sites

Topic Name: Chigga gets backpack.tf members to do his homework for him.

Eh, yeah it's my homework, but I need smart people to explain how to do the problems to me. And you guys are smart, right? <3

Link to comment
Share on other sites

 

Here's what I got:

#1 I= P/V
      = 0.25W/6V
      = 0.0417A
   
#2 V^2= PR 
          = 0.25W*1200ohm
          = 300V^2 
     V= 17.3V
 
#3 I^2= P/R 
         = 0.125W / 1200ohm
         = 0.000104 A^2
     I= 0.01A

 

Someone please confirm ^^^

 

edit: 

for #2, wouldn't it be 0.5W*1200ohm?

final answer ~ 24.497V

 

 

Thanks crannybabe <3

Link to comment
Share on other sites

Archived

This topic is now archived and is closed to further replies.

  • Recently Browsing   0 members

    • No registered users viewing this page.
×
×
  • Create New...